The number of cars sold at a dealership over several weeks is given below.

14, 23, 31, 29, 33

What is the standard deviation for this set of population data?

Standard deviation: Sigma = StartRoot StartFraction (x 1 minus mu) squared + (x 2 minus mu) squared + ellipsis + (x N minus mu) squared Over N EndFraction EndRoot
6.9
12.4
15.4
47.2

The Number Of Cars Sold At A Dealership Over Several Weeks Is Given Below.14, 23, 31, 29, 33What Is The

Answers

Answer 1

Answer:

A. 6.9

Step-by-step explanation:

Edge 2020

Answer 2

The standard deviation for the set of population data given in the question is 6.9

How to determine the meanData = 14, 23, 31, 29, 33Number of data (n) = 5Summation of data = 14 + 23 + 31 + 29 + 33 = 130Mean (μ) =?

Mean = summation of data / number

μ = 130 / 5

μ = 26

How to determine the standard deviationData = 14, 23, 31, 29, 33Mean (μ) = 26Number of data (n) = 5Standard deviation (σ) =?

σ = √[[(x₁ - μ)² + (x₂ - μ)² + (x₃ - μ)² + (x₄ - μ)² + (x₅ - μ)²] / n]

σ = √[[(14 - 26)² + (23 - 26)² + (31 - 26)² + (29 - 26)² + (33 - 26)²] / 5]

σ = √[236 / 5}

σ = 6.9

Learn more about statistics:

https://brainly.com/question/8974461

#SPJ9


Related Questions

What is the volume of this cube with a side length of 6 centimeters
6 cm

Answers

Answer:

V = 216 cm^3

Step-by-step explanation:

The volume of a cube is given by

V = s^3 where s is the side length

V = (6)^3

V = 216 cm^3

Answer:216 cm^3

Step-by-step explanation:

In cube, the length of all sides are equal

length of side=6cm

Volume of cube=length x length x length

Volume of cube=6 x 6 x 6

Volume of cube=216

Volume of cube=216 cm^3

9x-3=87 what is the anwser

Answers

Answer:

x=10

Step-by-step explanation:

9x-3=87

add 3 to both sides

9x=90

divide by 9 on both sides

x=10

Answer:

x=10

Step-by-step explanation:

9x-3=87

you add 3 to both sides

9x-3(+3)=87(+3)

which equals

9x=90

90/9= 10

answer:

x =10

I hope this helped!

Two samples each of size 20 are taken from independent populations assumed to be normally distributed with equal variances. The first sample has a mean of 43.5 and standard deviation of 4.1 while the second sample has a mean of 40.1 and standard deviation of 3.2. A researcher would like to test if there is a difference between the population means at the 0.05 significance level. What can the researcher conclude?

Answers

Answer:

[tex]t=\frac{(43.5 -40.1)-(0)}{3.678\sqrt{\frac{1}{20}+\frac{1}{20}}}=2.923[/tex]

The degrees of freedom are

[tex]df=20+20-2=38[/tex]

And the p value is given by:

[tex]p_v =2*P(t_{38}>2.923) =0.0058[/tex]

Since the p value for this cae is lower than the significance level of 0.05 we have enough evidence to reject the null hypothesis and we can conclude that the true means for this case are significantly different

Step-by-step explanation:

When we have two independent samples from two normal distributions with equal variances we are assuming that  

[tex]\sigma^2_1 =\sigma^2_2 =\sigma^2[/tex]

And the statistic is given by this formula:

[tex]t=\frac{(\bar X_1 -\bar X_2)-(\mu_{1}-\mu_2)}{S_p\sqrt{\frac{1}{n_1}+\frac{1}{n_2}}}[/tex]

Where t follows a t distribution with [tex]n_1+n_2 -2[/tex] degrees of freedom and the pooled variance [tex]S^2_p[/tex] is given by this formula:

[tex]S^2_p =\frac{(n_1-1)S^2_1 +(n_2 -1)S^2_2}{n_1 +n_2 -2}[/tex]

The system of hypothesis on this case are:

Null hypothesis: [tex]\mu_1 = \mu_2[/tex]

Alternative hypothesis: [tex]\mu_1 \neq \mu_2[/tex]

We have the following data given:

[tex]n_1 =20[/tex] represent the sample size for group 1

[tex]n_2 =20[/tex] represent the sample size for group 2

[tex]\bar X_1 =43.5[/tex] represent the sample mean for the group 1

[tex]\bar X_2 =40.1[/tex] represent the sample mean for the group 2

[tex]s_1=4.1[/tex] represent the sample standard deviation for group 1

[tex]s_2=3.2[/tex] represent the sample standard deviation for group 2

First we can begin finding the pooled variance:

[tex]\S^2_p =\frac{(20-1)(4.1)^2 +(20 -1)(3.2)^2}{20 +20 -2}=13.525[/tex]

And the deviation would be just the square root of the variance:

[tex]S_p=3.678[/tex]

The statistic is givne by:

[tex]t=\frac{(43.5 -40.1)-(0)}{3.678\sqrt{\frac{1}{20}+\frac{1}{20}}}=2.923[/tex]

The degrees of freedom are

[tex]df=20+20-2=38[/tex]

And the p value is given by:

[tex]p_v =2*P(t_{38}>2.923) =0.0058[/tex]

Since the p value for this cae is lower than the significance level of 0.05 we have enough evidence to reject the null hypothesis and we can conclude that the true means for this case are significantly different

Using the t-distribution, as we have the standard deviation for the sample, it is found that the researcher can conclude that there is a difference between the population means at the 0.05 significance level.

What are the hypothesis tested?

At the null hypothesis, it is tested if there is no difference, that is:

[tex]H_0: \mu_1 - \mu_2 = 0[/tex]

At the alternative hypothesis, it is tested if there is a difference, that is:

[tex]H_a: \mu_1 - \mu_2 \neq 0[/tex]

What is the mean and the standard error of the distribution of differences?

For each sample, we have that they are given by

[tex]\mu_1 = 43.5, s_1 = \frac{4.1}{\sqrt{20}} = 0.9168[/tex]

[tex]\mu_2 = 40.2, s_2 = \frac{3.2}{\sqrt{20}} = 0.7155[/tex]

Hence, for the distribution of differences, the mean and the standard error are given by:

[tex]\overline{x} = \mu_1 - \mu_2 = 43.5 - 40.2 = 3.3[/tex]

[tex]s = \sqrt{s_1^2 + s_2^2} = \sqrt{0.9168^2 + 0.7155^2} = 1.163[/tex]

What is the test statistic?

It is given by:

[tex]t = \frac{\overline{x} - \mu}{s}[/tex]

In which [tex]\mu = 0[/tex] is the value tested at the null hypothesis, hence:

[tex]t = \frac{\overline{x} - \mu}{s}[/tex]

[tex]t = \frac{3.3 - 0}{1.163}[/tex]

[tex]t = 2.84[/tex]

What is the decision?

Considering a two-tailed test, as we are testing if the mean is different of a value, with a significance level of 0.05 and 20 + 20 - 2 = 38 df, the critical value is of [tex]|z^{\ast}| = 2.0244[/tex].

Since the absolute value of the test statistic is greater than the critical value, it is found that the researcher can conclude that there is a difference between the population means at the 0.05 significance level.

More can be learned about the t-distribution at https://brainly.com/question/16313918

Determine if the set of vectors shown to the right is a basis for IR3 If the set of vectors is not a basis, determine whether it is linearly independent and whether the set 311-4 spans R 12 Which of the following describe the set?
A. The set is a basis for R3
B. The set is linearly independent.
C The set spans R3
D. None of the above

Answers

Answer:

The problem is clearly solved in the attachment

In ΔXYZ, the measure of ∠Z=90°, the measure of ∠X=57°, and XY = 8 feet. Find the length of YZ to the nearest tenth of a foot.

Answers

Answer:

21

Step-by-step explanation:

Answer:

6.7

Step-by-step explanation:

Sally wants to fill ten 8-inch tea glasses. How much tea does she need?
A) 80 ounces
B) 40 ounces
C) 80 cubic inches
D) Not enough information to answer

Answers

Answer

I believe C considering cubic mass

Answer:

D

Step-by-step explanation:

will mark the branliest to first one who answers

Answers

Answer:

3 1/4

Step-by-step explanation:

3/4 + (1/3 ÷1/6) - (-1/2)

Subtracting a negative is adding

3/4 + (1/3 ÷1/6) +1/2

Parentheses first

Copy dot flip

3/4 + (1/3 * 6/1) +1/2

3/4 + 2 + 1/2

Get a common denominator

3/4 + 2 + 2/4

2 + 5/4

2 + 4/4 +1/4

2+1 + 1/4

3 1/4

What is the area of the triangle?
PLSSS help me

Answers

Answer:

The area of the triangle is [tex]A=6 \:units^2[/tex].

Step-by-step explanation:

The area A of a triangle is given by the formula [tex]A=\frac{1}{2} bh[/tex] where b is the base and h is the height of the triangle.

From the graph, we can see that the base is 3 units and the height is 4 units. Therefore, the area of the triangle is

[tex]A=\frac{1}{2} \cdot3\cdot 4=\frac{12}{2}=6 \:units^2[/tex]

Need help with this math problem

Answers

Answer:

[tex]f(x)=-5x-3[/tex].

Step-by-step explanation:

From the given machine diagram it is clear that:

[tex]f(x)=-8[/tex] at [tex]x=1[/tex]

[tex]f(x)=-13[/tex] at [tex]x=2[/tex]

[tex]f(x)=-18[/tex] at [tex]x=3[/tex]

It is clear that the value of f(x) decreasing by 5 when the value of x is increasing by 1.

Since the function changing at a constant rate, therefore it represents a linear function.

If a linear function passing through two points, then the equation of line is

[tex]y-y_1=\dfrac{y_2-y_1}{x_2-x_1}(x-x_1)[/tex]

The given linear function passes through (1,-8) and (2,-13), therefore the linear equation is

[tex]y-(-8)=\dfrac{-13-(-8)}{2-1}(x-1)[/tex]

[tex]y+8=\dfrac{-5}{1}(x-1)[/tex]

[tex]y+8=-5(x-1)[/tex]

[tex]y=-5x+5-8[/tex]

[tex]y=-5x-3[/tex]

So, the required function is [tex]f(x)=-5x-3[/tex].

A slot machine has 3 dials each dial has 30 positions one of which is jackpot. To win jackpot all three dials must be in jackpot position. Assuming each play spins the dials and stops each independently and randomly, what are the odds of one play winning the jackpot

Answers

Answer:

3/90

Step-by-step explanation:

1 slot  is 1/30

2 slot is 1/30

3 slot is 1/30

this gives you that 3/90 when you had them

Answer:

D)  1/(30×30×30) = 1/27000 = 0.00003 or 0.003%

Step-by-step explanation:

e is 5 more than d.
fis 7 less than d.
a) Write an expression for e in terms of d.

Answers

Answer:

C

Step-by-step explanation:

Answer:

e = d + 5

Step-by-step explanation:

e = d + 5

f = d - 7

Solve for e means write as e = ...d

and this is already there...

You can not write it more compact then this.

If you try, you will notice you finally end with the initial equation which you started with, or you endup with something which is obviously very true like

e = e or d = d.

3(12−5)+(8x8)-45? Answer?

Answers

Answer:

its 40

Step-by-step explanation:

i think

40
Pemdas
Ones you solve what’s in the parenthesis it leaves you with 3x7+64-45 now you do the multiplication which leaves you with 21+64-45 ones you add it gives you 85-45 and now you have 40

A fraction that is equivalent to 6/-5?

Answers

Answer:

12/-10

Step-by-step explanation:

Any multiple of a fraction is the equivalent of the original fraction, the only difference is that it wont be fully simplified. If we multiply the original fraction (6/-5) by 2, both the numerator and denominator, you will get 12/-10.

Answer:

12/-10

Step-by-step explanation:

6/-5

6×2= 12

-5×2=-10

12/-10

Which value has an absolute deviation of 5 from the mean of this data set?
26, 12, 35, 28, 14
A 28
B. 35
C. 26
D. 14

Answers

Answer: 28

Step-by-step explanation: see prev. explanation

The absolute deviation of 5 from the mean of this data set is 28.

What is absolute deviation?

Absolute deviation is "the distance between each data point to the mean".

According to the question,

The data set is 26, 12, 35, 28, 14

Average of the data set = [tex]\frac{sum of the data value }{Total number of observation}[/tex]

= [tex]\frac{26+12+35+28+14}{5}[/tex]

= [tex]\frac{115}{5}[/tex]

= 23.

Thus, the average of the data set is 23.

In order to find absolute deviation of 5 subtract each data point from the mean.

26 - 23 = |3| = 3

12 - 23  = |-11| = 11

35 - 23 = |12| = 12

28 - 23 = |5| =  5

14 - 23  = |-9| = 9.

Hence, the absolute deviation of 5 is from the mean of the data set is 28.

Learn more about absolute deviation here

https://brainly.com/question/4364130

#SPJ2

Which fraction is in simplest form 4/20 6/9 5/13 14/21

Answers

Answer: 5/13

Step-by-step explanation:

Answer:

5/13 is in simplest form, because it cannot be reduced any further.

Step-by-step explanation: 4/20 can be reduced to 1/5, 6/9 to 1/3, and 14/21 can be reduced to 2/3

Employees that work at a fish store must measure the level of nitrites in the water each day. Nitrite levels should remain lower than 5 ppm as to not harm the fish. The nitrite level varies according to a distribution that is approximately normal with a mean of 3 ppm. The probability that the nitrite level is less than 2 ppm is 0.0918.
1. Which of the following is closest to the probability that on a randomly selected day the nitrite level will be at least 5 ppm?
(A) 0.0039
(B) 0.0266
(C) 0.0918
(D) 0.7519
(E) 0.9961

Answers

Answer: .0039

Step-by-step explanation:

100 POINTS

PLEASE PROVIDE STEPS.

THANK YOU!!!

Answers

Answer:

⅓ m/s

Step-by-step explanation:

Area of a square is:

A = s²

Take derivative of both sides with respect to time:

dA/dt = 2s ds/dt

Given that dA/dt = 6 m²/s and s = 9 m:

6 m²/s = 2 (9 m) ds/dt

ds/dt = ⅓ m/s

The Formula of the area of a square: A = bh or A = s^2

Solution:

~Take derivative of both sides

da/dt = 2s * ds/dt

~Use given values (6m^2/s and 9m)

6 = 2(9) * ds/dt

~Simplify

1/3m/s = ds/dt

Best of Luck!

10 X 5/11=

just multiplication get it right and get brainlyest
60pts or whatever it narrows it down to

Answers

The answer is 4.55

When you divide 5 by 11 you will get  0.45454545454 than when you multiply these numbers by 10 you will get 4.54545454545 and then the rounded answer is 4.55.

Answer: 4.54

Step-by-step explanation:

Drag each tile to the correct box. Not all tiles will be used.
Arrange the equations in the correct sequence to find the inverse of f(x) = y = 3x / 8 + x​

Answers

Answer:

Inverse of f(x)

               [tex]f^{l} (x) = \frac{8 x}{3-x}[/tex]

Step-by-step explanation:

Explanation:-

Step(i):-

Given the function

                        [tex]f(x) = \frac{3 x}{8+x}[/tex]

Given function is one-one and onto function

Hence f(x) is bijection function

                   [tex]y = f(x) = \frac{3 x}{8+x}[/tex]

now cross multiplication, we get

            ( 8+x)y = 3 x

             8 y + x y = 3 x

             8 y = 3 x - x y

taking Common 'x' we get

            x (3 - y) = 8 y

                   [tex]x = \frac{8 y}{3-y}[/tex]

Step(ii):-

The inverse function

                 [tex]x = \frac{8 y}{3-y} = f^{l}(y)[/tex]

The inverse function of x

                  [tex]f^{l}(x) = \frac{8 x}{3-x}[/tex]

Final answer:-

Inverse of f(x)

               [tex]f^{l} (x) = \frac{8 x}{3-x}[/tex]

what is the cube root of 1

Answers

Answer:

1.

Step-by-step explanation:

∛1= 1.

It can also be seen as:

1×1×1= 1.

Apply the distributive property to factor out the greatest common factor of all three terms. Explanation: 9-12x+6y what is the answer??

Answers

Answer: [tex]3(3-4x+2y)[/tex]

Step-by-step explanation:

[tex]9-12x+6y[/tex]

[tex]3(3-4x+2y)[/tex]

Graph the circle (x-3)^2+(y-7)^2=4

Answers

The equation is in the form of the equation of the circle. x^2+y^2=r^2. This equation is for the centre, (0,0).

x-3 and y-7 represents the shift in the centre of the circle in the respective axis. Therefore the centre is (3,7). r=2. d=4. So this circle’s centre is at (3,7) with radius 2.

A music professor offers his 40 students the option of coming to an additional rehearsal session the week before their juries (musical final exams.) In order to decide whether these extra sessions actually help students, he keeps track of who attends them and compares their jury scores to those of students who did not schedule extra sessions. This study is a(n): A) matched pairs design. B) randomized block design. C) nonrandomized experiment. D) observational study. E) completely randomized experiment.

Answers

Answer:

D. Observational Study

Explanation:

An observational  study is one in which all the participants are subjected to a common treatment and then compared to people who did not receive the same treatment. This is the case with the students who where subjected to the same treatment; an additional rehearsal session. They are then observed by the professor and compared to those who did not participate in the experiment.  

This is also an example of a cohort observational study. A cohort observational study is one in which all the participants have a common uniting factor. They are made to undergo a treatment and then compared to those who did not receive the treatment. This type of study  is subject to bias because a positive or negative result might be because of other factors not related to the study.

Suppose you had to
guess on a four-choice
multiple-choice test and
were given four questions.
Find the binomial
probability distribution.
( + ) ℎ =
4 = 0.25

Answers

Answer:

For 0 correct answer [tex]^4c_0p^0q^{4-0}[/tex]

For 1 correct answer [tex]^4c_1p^1q^{4-1}[/tex]

For 2 correct answer [tex]^4c_2p^0q^{4-2}[/tex]

For 3 correct answer [tex]^4c_3p^1q^{4-3}[/tex]

For 4 correct answer [tex]^4c_4p^1q^{4-4}[/tex]

Step-by-step explanation:

It is given that there are 4 questions n = 4

Number of choices is 4

So probability of getting correct answer [tex]=\frac{1}{4}[/tex]

Probability of getting incorrect answer [tex]=1-\frac{1}{4}=\frac{3}{4}[/tex]

Probability distribution is given by [tex]^nc_rp^rq^{n-r}[/tex]

Therefore probability distribution of 0 correct answer

[tex]^4c_0p^0q^{4-0}[/tex]

Therefore probability distribution of 1 correct answer

[tex]^4c_1p^1q^{4-1}[/tex]

Therefore probability distribution of 2 correct answer

[tex]^4c_2p^0q^{4-2}[/tex]

Therefore probability distribution of 3 correct answer.

[tex]^4c_3p^1q^{4-3}[/tex]

Therefore probability distribution of 4 correct answer.

[tex]^4c_4p^1q^{4-4}[/tex]

Please help ASAP! Will give BRAINLIEST! Please read the question THEN answer correctly! No guessing.

Answers

Answer:

B

Step-by-step explanation:

Answer:

B

Step-by-step explanation:

B because first you need to read the problem and understand the information.

perform the following operations on matrices (1 8 0 7) (7 6 7 4) = ( )

Answers

Answer:

  [tex]\left[\begin{array}{cc}63&38\\49&28\end{array}\right][/tex]

Step-by-step explanation:

  [tex]\left[\begin{array}{cc}1&8\\0&7\end{array}\right]\left[\begin{array}{cc}7&6\\7&4\end{array}\right]=\left[\begin{array}{cc}(1)(7)+(8)(7)&(1)(6)+(8)(4)\\(0)(7)+(7)(7)&(0)(6)+(7)(4)\end{array}\right]\\\\=\left[\begin{array}{cc}63&38\\49&28\end{array}\right][/tex]

Each element of the product matrix is the dot product of the corresponding row in the left matrix and the corresponding column in the right matrix.

For example, the element at row 2, column 1 of the product is [0 7]·[7, 7], the dot product of row 2 of the left matrix with column 1 of the right matrix.

_____

Many calculators, spreadsheets, and web sites can do this tedious math for you.

Junior bought a bag of mixed fruit snacks. The flavors in the bag are 4 strawberry, 3 cherry, and 5 grape. If he chooses one fruit snack at random, what it the probability of the first one being grape?

Answers

Answer:I believe it would be 5/12

Step-by-step explanation:

You add all of them up then since it's 5 grapes and in total there is 12 fruit snacks. It should be 5 grapes of 12 fruit snacks in the bag.

Brittany Monroe is a legal secretary. Her biweekly salary is $1,650.00 what is her annual salary?

Answers

Answer:

$42,900 a year

Step-by-step explanation:

so there are 26 bi-weeks in a year. (fun fact)

you take $1,650 and multiply that biweekly to get her annual salary.

1650*26=42,900

A company makes car batteries and claims 80% of its ABC batteries are good for 70 months or longer. Assume that this claim is true. Let p ˆ be the proportion in a sample of 100 such ABC batteries. What is the probability that this sample proportion is within 0.05 of the population proportion.

Answers

Answer:

78.88% probability that this sample proportion is within 0.05 of the population proportion

Step-by-step explanation:

We need to understand the normal probability distribution and the central limit theorem to solve this question.

Normal probability distribution

Problems of normally distributed samples are solved using the z-score formula.

In a set with mean [tex]\mu[/tex] and standard deviation [tex]\sigma[/tex], the zscore of a measure X is given by:

[tex]Z = \frac{X - \mu}{\sigma}[/tex]

The Z-score measures how many standard deviations the measure is from the mean. After finding the Z-score, we look at the z-score table and find the p-value associated with this z-score. This p-value is the probability that the value of the measure is smaller than X, that is, the percentile of X. Subtracting 1 by the pvalue, we get the probability that the value of the measure is greater than X.

Central Limit Theorem

The Central Limit Theorem estabilishes that, for a normally distributed random variable X, with mean [tex]\mu[/tex] and standard deviation [tex]\sigma[/tex], the sampling distribution of the sample means with size n can be approximated to a normal distribution with mean [tex]\mu[/tex] and standard deviation [tex]s = \frac{\sigma}{\sqrt{n}}[/tex].

For a skewed variable, the Central Limit Theorem can also be applied, as long as n is at least 30.

For proportion p in a sample of size n, we have that [tex]\mu = p, s = \sqrt{\frac{\pi(1-\pi)}{n}}[/tex]

In this question:

[tex]p = 0.8, n = 100[/tex]

So

[tex]\mu = 0.8, s = \sqrt{\frac{0.8*0.2}{100}} = 0.04[/tex]

What is the probability that this sample proportion is within 0.05 of the population proportion.

This is the pvalue of Z when X = 0.8 + 0.05 = 0.85 subtracted by the pvalue of Z when X = 0.8 - 0.05 = 0.75.

X = 0.85

[tex]Z = \frac{X - \mu}{\sigma}[/tex]

By the Central Limit Theorem

[tex]Z = \frac{X - \mu}{s}[/tex]

[tex]Z = \frac{0.85 - 0.8}{0.04}[/tex]

[tex]Z = 1.25[/tex]

[tex]Z = 1.25[/tex] has a pvalue of 0.8944.

X = 0.75

[tex]Z = \frac{X - \mu}{s}[/tex]

[tex]Z = \frac{0.75 - 0.8}{0.04}[/tex]

[tex]Z = -1.25[/tex]

[tex]Z = -1.25[/tex] has a pvalue of 0.1056.

0.8944 - 0.1056 = 0.7888

78.88% probability that this sample proportion is within 0.05 of the population proportion

Find two numbers for which the sum is 101 and the difference is 47

Answers

Answer:

74 and 27

Step-by-step explanation:

let x and y be the numbers

x + y =101........eqn 1

x - y = 47.......eqn 2

solve simultaneously

from equation 2, make x the subject

x= 47 + y........eqn 3

put eqn 3 into eqn 1

(47+y) + y = 101

47 + 2y = 101

       2y= 101 - 47

       2y=54

         y= 54/2

         y= 27

put y=27 into eqn 3

x = 47 + 27

x = 74

74 and 27

74+27=101
74–27=47

Plzzz mark as brainleast plzzz plzzz
Other Questions
What body part makes the insulin that your body needs? What is the measure of STR Find the value of x and y. WILL MARK BRAINLIEST The smallest multiple of 13 is Goshford Company produces a single product and has capacity to produce 105,000 units per month. Costs to produce its current sales of 84,000 units follow. The regular selling price of the product is $126 per unit. Management is approached by a new customer who wants to purchase 21,000 units of the product for $77.40 per unit. If the order is accepted, there will be no additional fixed manufacturing overhead and no additional fixed selling and administrative expenses. The customer is not in the companys regular selling territory, so there will be a $7.60 per unit shipping expense in addition to the regular variable selling and administrative expenses. Per Unit Costs at 84,000 Units Direct materials $ 12.50 $ 1,050,000 Direct labor 15.00 1,260,000 Variable manufacturing overhead 14.00 1,176,000 Fixed manufacturing overhead 17.50 1,470,000 Variable selling and administrative expenses 14.00 1,176,000 Fixed selling and administrative expenses 13.00 1,092,000 Totals $ 86.00 $ 7,224,000 Calculate the combined total net income if the company accepts the offer to sell additional units at the reduced price of $77.40 per unit. the winter of 1977 turned out to be coldest in. years Ben makes $39,600 a year. What the maximum amount he can afford for a mortgage each month ? Which of the following represents an example of synthesis?Creons change of heart toward the burial of PolyniecesOdysseuss deception of the CyclopsPerseuss rescue of AndromedaJunos transformation of Io A CONE IS 1/3 OF A CYLINDER, SO THE VOLUME FORMULA IS 1/3 THE CYLINDER. What is the approximate area of a circle with diameter 6? PLEASE HURRY !Choose the item that best matches the verb givensentarsela navajael saco de dormirla butacael espejola camisael champel cepillo para los dientesel nombreNetel cerealla pastilla de jabn What does the first person narrative reveal about the setting? Jayna recently participated in a study at the local university hospital. As she was waiting in the study's waiting room the staff approached her and apologized for the temperature in the room. She was informed that the temperature of the waiting room would continue to increase because the thermostat was broken and was scheduled to be fixed later that week. Researchers provided her with cold beverages and warm beverages as she waited and had her complete some measures while she was waiting to be called. Little did Jayna know that she was already part of the study. There was nothing wrong with the waiting room's temperature and researchers monitored the numbers of beverages she drank and found that she only had cold beverages and noted multiple times how hot and uncomfortable the room was. Jayna was most likely in the: Can someone explain to me how to do itShow me an example please? Match the time period with the correct layer of fossils (mya = millions of years ago). ITEM BANK: Move to Bottom 175 mya250 mya400 mya 175 mya 0 2.25x = 1.5x + 6What is the value of x Write two sentences on global warming how wwl was connected to wwll Based on the information in the chart, which three statements reflect advantages that the North had over the South in terms of winning the Civil War?a. the North had the ability to manufacture weaponsb. the South did not have as great a means of transportationc. the North had fewer urban centersd. the South did not have the ability to feed their troopse. the South was not as economically diverse What is the slope line through (-5,-10) and (-1,5)